LSAT and Law School Admissions Forum

Get expert LSAT preparation and law school admissions advice from PowerScore Test Preparation.

 lsatstudier
  • Posts: 49
  • Joined: Oct 24, 2016
|
#31997
Hi,

Why is the answer to this question B and not A? Where in the passage does it talk about this? I found where it talks about Answer A but not Answer B.

Thank you!
 Kristina Moen
PowerScore Staff
  • PowerScore Staff
  • Posts: 230
  • Joined: Nov 17, 2016
|
#32037
Hi lsatstudier,

To start with, this question asks about "the behavior of A. aperta in conflict situations." So we want to find that section in the passage. The actual behavior is described in the second paragraph.

We find out that behavior varies from contest to contest. Then we're told a theory that might help to explain why, and that is evolutionary game theory, where strategies are weighed to maximize payoff. The last paragraph contains predictions based on the theory.

Check out Line 30: "For example, a spider may engage in escalated fighting during a dispute only if the disputed resource is valuable enough to warrant the risk of physical injury. " That supports answer choice (B).

It would help if you could direct me to the section or line that you feel supports answer choice (A), and we can discuss it further.
 J1445
  • Posts: 11
  • Joined: Aug 05, 2020
|
#79042
Hi,

I am still confused about why B is correct. We know that fighting behavior varies from contest to contest and that EGT can be a possible explanation. However, I feel like it's a leap to say that therefore it is possible that in one contest they will resort to display. Is this correct because of the word "may"? Thanks!
 ToluOj
  • Posts: 8
  • Joined: Jul 08, 2020
|
#79550
Hi,

I'd like to know why A is wrong as well. I'd narrowed my answers down to A v. B, but I ended up choosing A because of Riechert's predictions in paragraph 3. I know that paragraph 3 doesn't explicitly mention conflict situations, but Riechert does predict that "the markedly different levels of competition for web sites in the two habitats will affect the spiders' willingness to engage in escalated fighting" in territorial disputes (I read as being territorial disputes as being basically synonymous to conflict situations). Does the word "suggest" in the question stem imply that we should pick the answer choice that isn't as explicitly stated in the passage? It just seems like both A and B are correct which I know can't be the case, so I appreciate your insight on this.

Thanks,
Tolu
 Rachael Wilkenfeld
PowerScore Staff
  • PowerScore Staff
  • Posts: 1358
  • Joined: Dec 15, 2011
|
#79557
Hi Tolu and J

The key in answer choice (A) is the causal language. They exhibit this behavior BECAUSE of competition for suitable sites. Remember on the LSAT causal language is means that when you see the cause, you expect to see the effect every time. That sort of relationship isn't supported in answer choice (A). While the passage talks about competition for sites as a reason for the variation in behavior, it doesn't establish that's the only reason for it. In fact, the passage lists lots of other factors, such as size of opponent, that impact this behavior.

Answer choice (B) on the other hand is phrased in terms of probability. It says they "may" limit their behavior to displays if the risk of injury is high, and the value of the resource is low. This is fairly clearly talked about starting around line 30.

Hope that helps you both!
Rachael
 ToluOj
  • Posts: 8
  • Joined: Jul 08, 2020
|
#79815
Ok yes! This makes sense because the passage literally lists a number of alternate causes for the same effect. Thanks Rachael!

Tolu
User avatar
 LSAT4Life
  • Posts: 17
  • Joined: Aug 10, 2021
|
#90188
Hi,

Can you explain why (D) is incorrect? I thought that the spiders tend to engage in escalated fighting, but think that I might be making a unwarranted assumption by thinking it. The passage explains it is more a case-by-case determination or contest by contest.

The passage states a spider may engage in escalated fighting only if "the disputed resource is valuable enough to warrant the risk of physical injury" which is more in line with (B). However, I'd appreciate an explanation on what makes (D) incorrect.
User avatar
 evelineliu
PowerScore Staff
  • PowerScore Staff
  • Posts: 91
  • Joined: Sep 06, 2021
|
#90191
Hi LSAT4Life,

(D) is incorrect because it distorts the author's meaning. The passage says that level of fighting does vary from contest to contest, but the passage does not suggest whether aggressive or nonaggressive is more common.

Hope that makes sense!
Eveline

Get the most out of your LSAT Prep Plus subscription.

Analyze and track your performance with our Testing and Analytics Package.